Surface integral, spherical coordinates, earth

Click For Summary
The discussion focuses on calculating the surface area of the Earth above 50 degrees latitude North using spherical coordinates. The initial integral setup leads to an incorrect result of 0.321, while the expected answer is 0.117. A key correction is identifying that in spherical coordinates, the angle θ should range from 0 to 40 degrees, not from 50 to 90 degrees. This adjustment resolves the discrepancy in the calculated surface area. The conversation highlights the importance of correctly setting integral limits in spherical coordinate problems.
HmBe
Messages
45
Reaction score
0

Homework Statement



Find the surface area of the Earth (as a fraction of the total surface of the earth) that lies above 50 degrees latitude North.




Homework Equations



$$A = \int_R\sqrt{|\det(g)|}d\theta d\phi$$


The Attempt at a Solution



Hence I get

$$\int_0^{360} \int_{50}^{90} r^2 \sin(\theta) d\theta d\phi$$

Which gives 0.321 as the answer, which just isn't right. The actual answer I know should be 0.117, or

$$\frac{1-\sin(50)}{2}$$

I'm sure something must be wrong with my integral, but I can't figure out what.
 
Physics news on Phys.org
HmBe said:

Homework Statement



Find the surface area of the Earth (as a fraction of the total surface of the earth) that lies above 50 degrees latitude North.




Homework Equations



$$A = \int_R\sqrt{|\det(g)|}d\theta d\phi$$


The Attempt at a Solution



Hence I get

$$\int_0^{360} \int_{50}^{90} r^2 \sin(\theta) d\theta d\phi$$

Which gives 0.321 as the answer, which just isn't right. The actual answer I know should be 0.117, or

$$\frac{1-\sin(50)}{2}$$

I'm sure something must be wrong with my integral, but I can't figure out what.

In the northern hemisphere, the spherical polar coordinate \theta is equal to 90^{\circ} minus the latitude. Thus the correct limits of the \theta integral are 0 and 40^{\circ}.
 
  • Like
Likes 1 person
You're a hero, I have mental problems. Thank you so much.
 
Question: A clock's minute hand has length 4 and its hour hand has length 3. What is the distance between the tips at the moment when it is increasing most rapidly?(Putnam Exam Question) Answer: Making assumption that both the hands moves at constant angular velocities, the answer is ## \sqrt{7} .## But don't you think this assumption is somewhat doubtful and wrong?

Similar threads

  • · Replies 14 ·
Replies
14
Views
2K
Replies
3
Views
2K
  • · Replies 7 ·
Replies
7
Views
3K
  • · Replies 2 ·
Replies
2
Views
2K
  • · Replies 3 ·
Replies
3
Views
3K
  • · Replies 9 ·
Replies
9
Views
1K
  • · Replies 7 ·
Replies
7
Views
2K
  • · Replies 1 ·
Replies
1
Views
1K
  • · Replies 4 ·
Replies
4
Views
2K
  • · Replies 4 ·
Replies
4
Views
2K